BDMO National Junior 2018/6

Discussion on Bangladesh Mathematical Olympiad (BdMO) National
User avatar
nahin munkar
Posts:81
Joined:Mon Aug 17, 2015 6:51 pm
Location:banasree,dhaka
BDMO National Junior 2018/6

Unread post by nahin munkar » Tue Jan 08, 2019 1:16 pm

Given $8$ lines on a plane and no two of them are parallel. Prove that, at least two of them form an angle less than 23°.
# Mathematicians stand on each other's shoulders. ~ Carl Friedrich Gauss

samiul_samin
Posts:1007
Joined:Sat Dec 09, 2017 1:32 pm

Re: BDMO National Junior 2018/6

Unread post by samiul_samin » Thu Jan 10, 2019 11:51 am

Using $PHP$

$23×8=184$
$184>180$
Which is not possible as two lines cannot form angle more than $180$ degree.
So,at least two of them form an angle less than $23$ degree. [Proved]
Anyone please clarify my short solution.

User avatar
Enthurelxyz
Posts:17
Joined:Sat Dec 05, 2020 10:45 pm
Location:Bangladesh
Contact:

Re: BDMO National Junior 2018/6

Unread post by Enthurelxyz » Wed Jan 20, 2021 10:54 am

Let the $8$ lines are $l_1,l_2,\cdots ,l_8$. Take a point $O$ on the same plane. Now draw $8$ lines $L_i$ on point $O%$ such that $L_i \parallel l_i$.

Now, on point $O$, there are $8$ different angles created by $L_1,L_2,\cdots ,L_8$. So, there should be at least an angle less than $23$.

Let $\angle L_iOL_j < 23$ then $l_i , l_j$ create an angle less than $23$ as they create an equal angle with $\angle L_iOL_j$.
and miles to go before we sleep
and miles to go before we sleep

User avatar
Anindya Biswas
Posts:264
Joined:Fri Oct 02, 2020 8:51 pm
Location:Magura, Bangladesh
Contact:

Re: BDMO National Junior 2018/6

Unread post by Anindya Biswas » Sat Feb 06, 2021 10:41 pm

Enthurelxyz wrote:
Wed Jan 20, 2021 10:54 am
Let the $8$ lines are $l_1,l_2,\cdots ,l_8$. Take a point $O$ on the same plane. Now draw $8$ lines $L_i$ on point $O%$ such that $L_i \parallel l_i$.

Now, on point $O$, there are $8$ different angles created by $L_1,L_2,\cdots ,L_8$. So, there should be at least an angle less than $23$.

Let $\angle L_iOL_j < 23$ then $l_i , l_j$ create an angle less than $23$ as they create an equal angle with $\angle L_iOL_j$.
I have almost same argument, since those angles add up to $180^{\circ}$, then on average they are $\frac{180^{\circ}}{8}=22.5^{\circ}$, so in order to get such average, we must need at least one angle $\leq 22.5^{\circ}$
"If people do not believe that mathematics is simple, it is only because they do not realize how complicated life is."
John von Neumann

User avatar
Mehrab4226
Posts:230
Joined:Sat Jan 11, 2020 1:38 pm
Location:Dhaka, Bangladesh

Re: BDMO National Junior 2018/6

Unread post by Mehrab4226 » Sat Feb 06, 2021 11:01 pm

Anindya Biswas wrote:
Sat Feb 06, 2021 10:41 pm
since those angles add up to $180^{\circ}$, then on average they are $\frac{180^{\circ}}{8}=22.5^{\circ}$, so in order to get such average, we must need at least one angle $\leq 22.5^{\circ}$
This was a clever one! And easy to express too!
The Mathematician does not study math because it is useful; he studies it because he delights in it, and he delights in it because it is beautiful.
-Henri Poincaré

Post Reply